Inscription / Connexion Nouveau Sujet
Niveau Prepa (autre)
Partager :

Probabilités

Posté par
Hugo188
01-03-21 à 22:46

Bonjour,

voici un exercice sur lequel je bloque et j'aurai besoin d'aide :

Soit Y une v.a. telle que Y(\Omega)= et X de loi géométrique de paramètre p. On suppose que X et Y sont indépendantes. Pour tout k1, déterminer P{X-Y=k | X>Y}.


Ducoup de l'énnoncé on a que X(\Omega)=N* et P{X=i}=(1-p)^{i-1}p  et Y(\Omega)=N. Or je ne connais pas P{y=j}.

Je voulais utiliser la formule des probabilités totales pour avoir:
P{X-Y=k | X>Y}=\sum_{j=0}^{\infty }{ P(Y=j,X=k+j|X>Y)}
=\sum_{j=0}^{\infty }{ P(Y=j)P(X=k+j)}  car X et Y sont indépendants mais apres je suis bloqué.

Merci d'avance,
Bonne soirée

Posté par
DOMOREA
Probabilités Posté par Hugo188 02-03-21 à 10:29

bonjour,
p(X-Y=k|X>Y)=p(X=Y+k) or X et Y sont indépendants donc p(X=j+k/Y=j)=p(X=j+k)

ainsi p(X-Y=k|X>Y)=\sum_{j=0}^{+\infty} p(X=j+k)

Posté par
GBZM
re : Probabilités 02-03-21 à 11:08

Bonjour,

L'égalité P(X-Y=k\mid X>Y)=P(X=Y+k) est fausse. On a bien entendu P(X-Y=k\mid X>Y)=P(X=Y+k)/P(X>Y).

Hugo188, ton idée de départ était bonne : décomposer l'évènement \{X>Y\} en réunion disjointe des évènements \{X>Y\text{ et }Y=j\}=\{X>j\text{ et }Y=y\} pour j\in \N.

N'oublie pas une propriété importante de la loi géométrique, le fait qu'elle est sans mémoire.  Autrement dit, P(X=j+k\mid X>j)= P(X=k) pour tout entier k>0.

Posté par
Hugo188
re : Probabilités 02-03-21 à 11:09

Ok je vois alors dans ce cas la :

P(X-Y=k | X>Y) = \sum_{j=0}^{+\infty }{P(X=j+k)}= \sum_{j=0}^{+\infty }{p(1-p)^{j+k-1}}=(1-p)^{k-1}

Merci

Posté par
Ulmiere
re : Probabilités 02-03-21 à 11:36

***message modéré***non respect des règles du forum***surtout que plus haut l'erreur avait été signalée et que le post était pris en charge***merci de le comprendre***

Posté par
GBZM
re : Probabilités 02-03-21 à 11:41

Entre DOMOREA qui emmène sur une fausse piste et Ulmière qui fait l'exercice à la place du questionneur ...

Posté par
GBZM
re : Probabilités 02-03-21 à 11:45

J'ajouterais que vu les hypothèses (X de loi géométrique et Y indépendante de X), P(X>Y) ne peut pas être nul !

Posté par
Ulmiere
re : Probabilités 02-03-21 à 12:02

J'ai fait ça pour rattraper le coup, pour ne pas qu' Hugo188 pense avoir réussi l'exercice de cette manière. Je voulais aussi attirer son attention sur le conditionnement.

Dans le cas dégénéré où p = 0 (il n'est pas précisé que 0<p<1), que se passe-t-il ?
Aussi, Y(\Omega) = \mathbb{N} nous dit que Y prend toutes (et uniquement des) valeurs entières, mais on ne connait pas sa loi, ça peut très bien être avec proba nulle en général. Ici évidemment c'est pas le cas parce que Y est définie sur le même espace que X, qui est une variable discrète.
Je crois que c'est un bon réflexe de toujours se poser des questions avant d'agir.

Posté par
GBZM
re : Probabilités 02-03-21 à 12:07

J'avais déjà signalé que l'indication donnée par DOMOREA était fausse et j'étais sur le point de le répéter. Là-dessus, tu viens faire l'exercice à la place du questionneur et tu insistes sur un problème qui n'en est pas un, malgré tes arguties.

Posté par
Ulmiere
re : Probabilités 02-03-21 à 12:36

Point d'argutie ici. Tu peux ronger ton frein et me reprocher d'avoir donné une solution toute faite si ça te chante, mais certainement pas de mettre le doigt sur un faux problème. C'est le problème de la division par zéro qui se pose ici, et si je ne le soulève pas, il passe totalement inaperçu. C'est une précision qui n'a aucun d'intérêt pour toi et moi qui sommes des professionnels, mais qui restera je l'espère, dans un coin de la tête du questionneur lorsqu'il passera ses concours.

Posté par
Hugo188
re : Probabilités 02-03-21 à 13:35

Ducoup vu qu'elle est sans mémoire cela devient immédiat je pense (même pas besoin de décomposer l'évènement?),
on a P(X-Y=k | X>Y)=P(X=k)=p(1-p)^{k-1}

Mais si on veut repartir de la :
P(X-Y=k | X>Y)=\frac{P(X=Y+K)}{P(X>Y)}
alors je ne vois pas trop comment calculer le terme au numérateur car on a alors
P(X-Y=k)=\sum_{j=0}^{\infty }{P(X=j+k,Y=j)}

Au dénominteur on a:
P(X>Y)=\sum_{j=0}^{\infty }{P(X>Y,Y=j)}=\sum_{j=0}^{\infty }{P(X>j)}=\sum_{j=0}^{\infty }{(1-p)^{j}}=\frac{1}{1-p}

Posté par
Hugo188
re : Probabilités 02-03-21 à 13:39

Au dénominateur c'est 1/p pardon

Posté par
GBZM
re : Probabilités 02-03-21 à 13:40

Hugo188 @ 02-03-2021 à 13:35

Ducoup vu qu'elle est sans mémoire cela devient immédiat je pense (même pas besoin de décomposer l'évènement?),
on a P(X-Y=k | X>Y)=P(X=k)=p(1-p)^{k-1}

Et où dans cet argument utilises-tu l'indépendance de X et Y ? Tu vois bien que ça ne va pas si tu prends par exemple Y=\lfloor X/2\rfloor.

Posté par
GBZM
re : Probabilités 02-03-21 à 13:45

Hugo188 @ 02-03-2021 à 13:35


\sum_{j=0}^{\infty }{P(X>Y,Y=j)}=\sum_{j=0}^{\infty }{P(X>j)}


De quoi de quoi ?

Posté par
GBZM
re : Probabilités 02-03-21 à 13:48

Ulmiere @ 02-03-2021 à 12:36

C'est le problème de la division par zéro qui se pose ici

Non puisqu'il est clair que P(X>Y) n'est pas nul vu les hypothèses. C'est d'ailleurs l'énoncé qui introduit la probabilité conditionnelle et demande de la calculer.

Posté par
Hugo188
re : Probabilités 02-03-21 à 14:07

GBZM @ 02-03-2021 à 13:45

Hugo188 @ 02-03-2021 à 13:35


\sum_{j=0}^{\infty }{P(X>Y,Y=j)}=\sum_{j=0}^{\infty }{P(X>j)}


De quoi de quoi ?


P(X>Y,Y=j)=P(X>j,Y=j)=P(X>j)P(Y=j) car X et Y sont indépants
et P(X>j)=(1-p)^{j}

Posté par
Hugo188
re : Probabilités 02-03-21 à 14:16

GBZM @ 02-03-2021 à 13:40

Hugo188 @ 02-03-2021 à 13:35

Ducoup vu qu'elle est sans mémoire cela devient immédiat je pense (même pas besoin de décomposer l'évènement?),
on a P(X-Y=k | X>Y)=P(X=k)=p(1-p)^{k-1}

Et où dans cet argument utilises-tu l'indépendance de X et Y ? Tu vois bien que ça ne va pas si tu prends par exemple Y=\lfloor X/2\rfloor.


P(X-Y=k | X>Y) = \sum_{j=0}^{\infty }{P(X=Y+k | X>Y , Y=j)}=\sum_{j=0}^{\infty }{P(X=k , Y=j)}=\sum_{j=0}^{\infty }{P(X=k )P( Y=j)}=P(X=k)\sum_{j=0}^{\infty }{P( Y=j)}=P(X=k)

mais ducoup avec votre contre exemple je vois bien qu'il y a un problème mais je ne comprends pas ou.

Posté par
GBZM
re : Probabilités 02-03-21 à 14:39

Euh... D'où sors-tu ta première égalité ?

Posté par
Hugo188
re : Probabilités 02-03-21 à 14:41

De la formule des probabilités totales.
Ce n'est pas bon?

Posté par
GBZM
re : Probabilités 02-03-21 à 14:45

Tu es vraiment sûr que si B est la réunion disjointe des B_i, alors P(A\mid B)= \sum_i P(A\mid B_i) ???

Posté par
Hugo188
re : Probabilités 02-03-21 à 14:54

GBZM @ 02-03-2021 à 14:45

Tu es vraiment sûr que si B est la réunion disjointe des B_i, alors P(A\mid B)= \sum_i P(A\mid B_i) ???

Non ici c'est P(A)= \sum_i P(A\mid B_i)

mais j'aurais plutôt du écrire P(X-Y=k | X>Y) = \sum_{j=0}^{\infty }{P(Y=j,X=Y+k | X>Y )}

Posté par
GBZM
re : Probabilités 02-03-21 à 15:19

Mais relis-toi ! Tu as bien écrit

Hugo188 @ 02-03-2021 à 14:16

P(X-Y=k | X>Y) = \sum_{j=0}^{\infty }{P(X=Y+k | X>Y , Y=j)

Tu vois bien que ça ne va pas du tout.

Bon, revenons sur terre. On cherche à calculer P(A\mid B) en utilisant que B est la réunion disjointe des B_j.
On a cette formule-ci (elle est correcte, vérifie-le) :

P(A\mid B) = \sum_j (P(A\mid B_j)P(B_j\mid B)) .

Ici bien sûr A =\{X=Y+k\}, B=\{X>Y\} et B_j=\{X>Y\text{ et }Y=j\}.

Posté par
DOMOREA
Probabilités 02-03-21 à 16:49

@GBMZ
il me semble ma réponse était bonne car (X=Y+k) et (X>Y) sont indépendants, je l'avais vu tout de suite.
Pourquoi ? (Y<X) est une union disjointe de (Y=i) avec i<X

Pour simplifier l'écriture posons  (Y=X+k)=A et (Y=i)=Bi

P(A/U_iBi)= \frac{P(A\cap (U_iB_i))}{p(U_iB_i)} on développe l'intersection, on somme les p(A\cap B_i)
les événements A et Bi  étant indépendants  on obtient \sum P(A)p(B_i) on met P(A) en facteur et on retrouve p(A)P(U_iB_i)
qui donc se simplifie avec le dénominateur

Posté par
GBZM
re : Probabilités 02-03-21 à 17:34

DOMOREA @ 02-03-2021 à 16:49

@GBMZ
il me semble ma réponse était bonne car (X=Y+k) et (X>Y) sont indépendants, je l'avais vu tout de suite.
Pourquoi ? (Y<X) est une union disjointe de (Y=i) avec i<X


M'enfin, DOMOREA, c'est grossièrement faux !!! Tu vois bien que l'évènement {X=Y+k} est contenu dans l'évènement {X>Y} puisque k>0. Comme on n'a pas presque sûrement X>Y, c'est que P(X+Y=k\mid X>Y) > P(X+Y=k).

DOMOREA, tout le monde peut faire des erreurs. Mais c'est un peu pénible de devoir autant insister (comme dans un fil précédent sur les points d'accumulation) pour que tu finisses par t'apercevoir des erreurs que tu fais.

Posté par
GBZM
re : Probabilités 04-03-21 à 09:21

Dommage que ce fil finisse en queue de poisson après pas mal d'errements.

C'est pourtant simple.
Première étape :

P(X-Y=k\mid X>Y) = \sum_j P(X-Y=k\mid X>Y\text{ et }Y=j)\,P( X>Y\text{ et }Y=j\mid X>Y)

Pour éviter qu'ulmière ne râle, on ne somme que sur les entiers naturels j tels que P(Y=j)>0.
Deuxième étape : on utilise l'indépendance de X et Y.

P(X-Y=k\text{ et }Y=j)=P(X=j+k)P(Y=j)
P(X>Y\text{ et }Y=j)=P(X>j)P(y=j)

On arrive donc à

P(X-Y=k\mid X>Y) = \sum_j \dfrac{P(X=j+k)}{P(X>j)}\,P( X>Y\text{ et }Y=j\mid X>Y)

Troisième étape : le fait que la loi géométrique soit sans mémoire nous dit que

 \\ \dfrac{P(X=j+k)}{P(X>j)}=P(X=k)=(1-p)^{k-1}p

On conclut maintenant avec

P(X-Y=k\mid X>Y) = (1-p)^{k-1}p\,\sum_j P( X>Y\text{ et }Y=j\mid X>Y)=(1-p)^{k-1}p

Posté par
DOMOREA
Probabilités 07-03-21 à 11:31

bonjour,
Je reviens sur cet exercice qui m'a perturbé et pas seulement parce que GBMZ m'a fortement rabroué;
Remarque la lettre p est utilisée pour 3 probabilités distinctes bien qu'il y ait 3 univers.

La variable aléatoire X a \mathbb{N^*} pour domaine, pour Y c'est \mathbb{N}
Une probabilité p est définie sur \mathbb{N^*}\times \mathbb{N}{ avec l'hypothèse d'indépendance des deux lois

la question : k>0 ,déterminer p(X=Y+k/X>Y) m'a d'emblée étonné  car la condition X>Y n'apporte rien à la condition X=Y+k
contrairement aux questions classiques de probabilités conditionnelles.

On pourrait ainsi sans perdre en généralité définir un univers \Omega_2=\{(x,y)/ x\in \mathbb{N^*},y\in \mathbb{N}, x>y\}

Dans cet univers  X et Y sont représentés en totalité , L'indépendance de X et Y est préservé.
Ainsi nous avons toujours p(X=3,y=2)=p(X=3)x p(Y=2)
mais dans ce contexte p(X=Y+k/X>Y) qui est toujours égal à \frac{p((X=Y+k)\cap (X>Y))}{p(X>Y)}
égal à \frac{p(X=Y+k)}{p(X>Y)} puisque (X=Y+k)\subset X>Y) mais avec p(X>Y)=p(\Omega_2)=1
donc j'arrive à p(X-Y=k/X>Y)=p(X=Y+k)= \sum_{i=0}^{\infty}p(X=i+k)=q^{k-1}

j'ajoute une remarque pour GBMZ
Tu trouves comme résultat p(X=k) or p(X=k)=p(X=k/X>0) mais alors quid de p(X=k+1/X>1),...

j'ai peut-être encore dit des bêtises mais j'aimerais comprendre

Posté par
Vassillia
re : Probabilités 07-03-21 à 13:41

Bonjour,
Je crois que j'ai compris pourquoi tu t'embrouilles avec cet exercice.
Pour k>0, l'égalité P((X=Y+k) \cap (X>Y)) = P(X=Y+k) est vraie, il n'y a pas de débat là-dessus je pense. Tu en déduis que la condition n'apporte rien et c'est là où le raisonnement ne va plus à mon avis.

La probabilité conditionnelle sachant (X>Y) est une nouvelle mesure de probabilité définie comme suit :
f(\omega) = P(\omega)/P(X>Y) si \omega \in \Omega_2 et f(\omega)=0 sinon

Toi tu restreints le problème à \Omega_2 donc tu en déduis P(X>Y)=1. Certes mais ce n'est pas très convaincant comme démonstration, pourquoi donc on ne pourrait pas avoir X<Y d'après l'énoncé initial ?

Ton discours revient à : A inclus dans B donc on peut en déduire P(A|B)=P(A). Cela peut surprendre quand tu insistes sur cette position car on est d'accord dans ce cas de figures que P(A|B)=\frac{P(A)}{P(B)}, non ?

J'espère avoir éclairci la situation et je te laisse poursuivre la discussion avec GBZM sans m'en meler

Posté par
GBZM
re : Probabilités 07-03-21 à 13:50

DOMOREA @ 07-03-2021 à 11:31

j'ai peut-être encore dit des bêtises mais j'aimerais comprendre


Oui, je confirme : tu as encore dit des bêtises.

Il y a un seul univers \Omega avec une seule mesure de probabilité P dans l'histoire. On ne précise pas qui est \Omega, pas besoin. Et tu n'as pas l'air de bien comprendre ce qu'est une variable aléatoire.
La variable aléatoire X est une fonction de \Omega dans \N^*. Le domaine de X n'est pas \N^* comme tu l'affirmes ; \N^* est l'image de X.
La variable aléatoire Y est une fonction de \Omega dans \N.

Citation :
la question : k>0 ,déterminer p(X=Y+k/X>Y) m'a d'emblée étonné  car la condition X>Y n'apporte rien à la condition X=Y+k

C'est vraiment n'importe quoi. La question "Quelle est la probabilité de tirer un 6 au dé sachant qu'on tire un nombre pair" t'étonnerait aussi ? Comprends-tu vraiment ce qu'est une probabilité conditionnelle ?  Le reste de ton message, qui n'a pas grand sens, prouve que non.

Je te conseille un recyclage approfondi sur les bases des probabilités.

Posté par
GBZM
re : Probabilités 07-03-21 à 14:18

Quelques petites précisions.

J'ai déjà écrit qu'on n'a pas besoin ici de préciser qui est \Omega (c'est souvent le cas en probas).
Mais, si on y tient, on peut prendre un \Omega qui fait l'affaire pour tout l'exercice : \Omega=\N^*\times \N, avec pour X la première projection et pour Y la deuxième projection. De la sorte, l'évènement \{(i,j)\}\subset \Omega est \{X=i\text{ et }Y=j\}.
La mesure de probabilité sur \Omega est donnée par les lois de X et Y et le fait que ces variables aléatoires sont indépendantes : la probabilité de l'évènement \{(i,j)\}\subset \Omega est (1-p)^{i-1}p\,P(Y=j).

L'événement B=\{X>Y\} est \{(i,j)\in \N^*\times \N\mid i>j\}. On a P(B)>0,  Il est impossible que les restrictions X|_B et Y|_B soient indépendantes pour la mesure de proba sur B donnée par la probabilité conditionnelle (une autre grosse erreur dans le message de DOMOREA).

Posté par
DOMOREA
Probabilités 07-03-21 à 17:46

bon ok je m'étais aperçu de mon erreur avant de lire ton post car mon p(\Omega _2) n'est pas égal à 1

En revanche, j'ai l'impression que tu as mal lu mon post du 2/3/21 à 16h49 qui me parait donner une bonne solution.

Pour me persuader j'ai fait l'effort de travailler sur un exemple \Omega(X)=\{1,2,...,6\} avec respectivement les probabilités \p,pq,...pq^5}
pour Y j'ai choisi \Omega(Y)=\{0,1,...,6\} avec les probabilités  respectives \{0,1;0,2;0,2;0,3;0,1;0,1,0\}.

j'ai choisi k=3,  p(X=Y+3/X>Y)=\sum_{i=0}^3p(X=3+i/Y<3+i)=\sum_{i=0}^3 \frac{ p(X=3+i)\cap(Y<3+i)}{p(Y<3+i))}

le détail de me calculs m'ont bien donné par exemple pour le numérateur de  l'indice i=1    p((X=3+1)\cap(Y<3+1)) la somme suivante:
0,1pq^3+0,2pq^3+0,2pq^3+0,3pq^3= 0,8pq^3 or on divise par p(Y<3+i)=p(Y<4)=0,8

c'est bien ce j'ai écrit dans mon post pré-cité (Y<4 défini par union puis distribution de l'intersection,, puis factorisation par le facteur constant)
Conforté, j'ai repris le cas général et je retrouve bien p(X=k+Y/X>Y)=\sum_{i=0}^{\infty}p(X=k+i})=pq^{k-1}\times \frac{q}{1-q}=q^k

Posté par
GBZM
re : Probabilités 07-03-21 à 18:03

DOMOREA, tu t'obstines dans l'erreur.

Ton exemple n'est pas un exemple de loi géométrique.

Et le calcul que tu fais après être "conforté" donne un résultat FAUX. La première égalité que tu écris est toujours aussi fausse.

C'est pas mal n'importe quoi, ce que tu fais.

Posté par
DOMOREA
Probabilités 08-03-21 à 12:17

Je sais que j'ai tronqué la loi, c'est parce j'ai bricolé un graphique et je me suis arrêté à X=6 , j'aurais du écrire \Omega(X)=\mathbb{N*}={1,2,...\infty} la question n'est pas là. idem pour \Omega(Y) qui est \mathbb{N}

oui j'ai fais une coquille les proba pour X sont p, pq,pq²... cela commence à p et je sais que la suite est infinie !! il y a aussi une erreur d'exposant dans le résultat final.

mais le coeur du problème est ce qui suit:

Quand on calcule p(X=Y+k/y<X) on effectue une somme de probabilités du type p(X=Y_0+k/Y\in [[0;Y_0+k-1]]) or l'événement Y\in [[0;Y_0+k-1]]
est un ensemble d'événements élémentaires indépendants par hypothèse de l'événement X=Y_0+k

Il est certain que si les lois marginales d'une loi définie sur \mathbb{N^*}\times \mathbb{N} sont indépendantes alors les événements (X=j+k) et (Y\leq j+k-1)   sont indépendants.

cela dit à part le fait que j'ai fait une erreur dans l'exposant du résultat final  qui est donc pour moi p(X=Y+k/Y<X)= q^{k-1}.

je ne comprends pas en quoi ce que j'ai écrit est faux.

Si c'est faux alors éclaire moi une dernière fois avec précision et sans m'esquinter SVP et ensuite j'abandonne sans rancune.

Posté par
GBZM
re : Probabilités 08-03-21 à 15:22

DOMOREA @ 08-03-2021 à 12:17


Quand on calcule p(X=Y+k/y<X) on effectue une somme de probabilités du type p(X=Y_0+k/Y\in [[0;Y_0+k-1]]) or l'événement Y\in [[0;Y_0+k-1]]
est un ensemble d'événements élémentaires indépendants par hypothèse de l'événement X=Y_0+k

Que veux-tu dire précisément ? Que

\large P(X=Y+k\mid X>Y) = \sum_{y_0} P(X=y_0+k\mid Y <y_0+k)

où la somme porte sur les entiers naturels y_0 tels que P(Y <y_0+k) >0 ?
Eh bien c'est faux. Prends par exemple le cas où Y est constante égale à 0. Alors le terme de gauche de ta prétendue égalité vaut P(X=k) et le terme de droite vaut P(X\geq k). Une erreur de plus !

Citation :
cela dit à part le fait que j'ai fait une erreur dans l'exposant du résultat final  qui est donc pour moi p(X=Y+k/Y<X)= q^{k-1}.

C'est toujours aussi faux. Le résultat correct est

\large P(X=Y+k\mid X>Y) = P(X=k) = q^{k-1}p

J'en ai donné la démonstration plus haut, dans ce message : Probabilités

S'il te plaît, fais un effort pour ne pas t'obstiner dans l'erreur et comprendre une démarche correcte.

Posté par
DOMOREA
Probabilités 09-03-21 à 15:09

GBZM, comment dans ton post du 04/03/21 à 9h21, démontres-tu ce que tu sembles utiliser, à savoir:

                        \sum_jP(X>Y et Y=j/X>Y)=1 ?

Posté par
GBZM
re : Probabilités 09-03-21 à 15:31

Tout simplement, l'évènement \{X>Y\} est la réunion disjointe de la famille dénombrable d'évènements \{ X>Y\text{ et }Y=j\} pour j\in \N.

Posté par
DOMOREA
Probabilités 10-03-21 à 11:52

GBZM,ok  en reprenant mes calculs j'ai compris mon erreur,
j'ai remarqué que tu avais décrit (X>Y) par les sections Y=constante alors que j'avais choisi par sections X=constante. Mais ce n'était pas là le problème,car pour poursuivre le calcul j'ai été obligé par regroupement de me retrouver avec des sections Y=constante

Quand j'ai déterminé P(X>Y)=\sum_{i=0}^{\infty} q^i P(Y=i) le P de probabilité est devenu par inattention le paramètre p de la loi géométrique, ce qui a fait disparaitre  p du résultat.

ainsi c'est clair: P((X=Y+k) \cap (X>Y))=

\sum_{i=0}^{\infty} pq^{k-1} q^i P(Y=i)=

p q^{k-1}\sum_{i=0}^{\infty} q^i P(Y=i)=

pq^{k-1}\times P(X>Y)

merci d'avoir pris le temps de me signaler mes errements

Posté par
GBZM
re : Probabilités 10-03-21 à 13:23

Ouf !

On retrouve bien les trois ingrédients ; l'utilisation du système complet d'évènements  (\{Y=j\})_{j\in \N}, l'indépendance de X et Y (utilisée par exemple pour P(X=k+i\text{ et }Y=i)=P(X=k+i)P(Y=i)) et le caractère sans mémoire de la loi géométrique P(X=k+i)=P(X=k)P(X>i)).



Vous devez être membre accéder à ce service...

Pas encore inscrit ?

1 compte par personne, multi-compte interdit !

Ou identifiez-vous :


Rester sur la page

Inscription gratuite

Fiches en rapport

parmi 1675 fiches de maths

Désolé, votre version d'Internet Explorer est plus que périmée ! Merci de le mettre à jour ou de télécharger Firefox ou Google Chrome pour utiliser le site. Votre ordinateur vous remerciera !